LSAT and Law School Admissions Forum

Get expert LSAT preparation and law school admissions advice from PowerScore Test Preparation.

User avatar
 Dave Killoran
PowerScore Staff
  • PowerScore Staff
  • Posts: 5848
  • Joined: Mar 25, 2011
|
#94748
Complete Question Explanation
(The complete setup for this game can be found here: lsat/viewtopic.php?f=269&t=1940)

The correct answer choice is (E)

If J stocks aisle 3, then the lowest-numbered aisles the MKM block could stock would be 4-5-6. Because O must stock a higher-numbered aisle than the MKM block, the lowest-numbered aisle O could stock would be aisle 7. Thus, O cannot stock aisle 6 and answer choice (E) is correct.
 SherryZ
  • Posts: 124
  • Joined: Oct 06, 2013
|
#11983
Q6: I chose A because the condition 6 requires that one of the L must be on 9.
But the correct answer is E.
Could you explain it?


Thank you very much!

----Sherry
User avatar
 Dave Killoran
PowerScore Staff
  • PowerScore Staff
  • Posts: 5848
  • Joined: Mar 25, 2011
|
#11986
Hi Sherry,

Let me turn this one around on you. Since the question is a Cannot Be True question, we know that the condition in (A) cannot occur. What happens when "Jill stocks aisle 9" ?

And as you look at that, take another look at the rule that involves L--does it always have to be in aisle 9? Does an impossible result necessarily follow?

Please let me know what you think. Thanks!
 SherryZ
  • Posts: 124
  • Joined: Oct 06, 2013
|
#12063
Hi Dave,

Thank you for your reply! I re-read the condition 6, I think I misunderstood it. It says "Exactly one of L's aisles is an end aisle." I think "End Aisle" is supposed to mean "Aisle 1 or 9", not must be Aisle 9. Right?

Thank you very much! Looking forward to your reply!

Sincerely,
Sherry
User avatar
 Dave Killoran
PowerScore Staff
  • PowerScore Staff
  • Posts: 5848
  • Joined: Mar 25, 2011
|
#12070
Hi Sherry,

Yes, that is correct :-D It's 1 or 9, not just 9. If you look at the game again with this in mind, I think you will now find it a bit easier to solve this question.

Thanks!
 studyhelp20
  • Posts: 28
  • Joined: Dec 09, 2020
|
#82527
Hello Power Score Support Staff,

Could you please provide me with a complete explanation of the correct answer to this question? I don't fully grasp the correct answer from the previous answered responses. A more in detail response of the entire question would be helpful.

Thanks,
Brennan
User avatar
 Stephanie Oswalt
PowerScore Staff
  • PowerScore Staff
  • Posts: 803
  • Joined: Jan 11, 2016
|
#82537
Hi Brennan,

Thanks for the post! I am going to send this back to you first! :)

Here's what I'd like you to do:

1. Describe your approach to the game setup. Were you able to determine what type of game it is? What the variable sets are?

2. Were you able to diagram any of the rules, or make any inferences?

3. What about the prior explanations had confused you, and did you have any specific questions?

I am confident that you will get more out of our help if we start with an understanding of how you approached the game and what you found difficult about this question. :)

Thanks!

Get the most out of your LSAT Prep Plus subscription.

Analyze and track your performance with our Testing and Analytics Package.